Difference between revisions of "Airy Bi"

From specialfunctionswiki
Jump to: navigation, search
(Properties)
Line 17: Line 17:
 
<div class="mw-collapsible-content">
 
<div class="mw-collapsible-content">
 
<strong>Proof:</strong> Suppose that $y$ has the form
 
<strong>Proof:</strong> Suppose that $y$ has the form
$$y(z) = \displaystyle\int_{C} f(t)e^{zt} dt,$$
+
$$y(z) = \displaystyle\int_{C} f(t)e^{-zt} dt,$$
 
where $C$ is an as-of-yet undefined contour in the complex plane. Assuming that we may differentiate under the integral it is clear that
 
where $C$ is an as-of-yet undefined contour in the complex plane. Assuming that we may differentiate under the integral it is clear that
$$y''(z)=\displaystyle\int_{C} f(t)t^2 e^{zt} dt.$$
+
$$y''(z)=\displaystyle\int_{C} f(t)t^2 e^{-zt} dt.$$
 
Thus we plug this representation into the differential equation to get
 
Thus we plug this representation into the differential equation to get
$$(*) \hspace{35pt} y''(z)-zy(z) = \displaystyle\int_{C} (t^2-z)f(t)e^{zt} dt = 0.$$
+
$$(*) \hspace{35pt} y''(z)-zy(z) = \displaystyle\int_{C} (t^2-z)f(t)e^{-zt} dt = 0.$$
 
Now we integrate by parts to see
 
Now we integrate by parts to see
 
$$\begin{array}{ll}
 
$$\begin{array}{ll}
\displaystyle\int_{C} zf(t)e^{zt} dt &= \displaystyle\int_{C} f(t) \dfrac{d}{dt} e^{zt} dt \\
+
\displaystyle\int_{C} zf(t)e^{-zt} dt &= -\displaystyle\int_{C} f(t) \dfrac{d}{dt} e^{-zt} dt \\
&= -f(t)e^{zt} \Bigg |_{C} + \displaystyle\int_{C} f'(t)e^{zt} dt.
+
&= -f(t)e^{zt} \Bigg |_{C} + \displaystyle\int_{C} f'(t)e^{-zt} dt.
 
\end{array}$$
 
\end{array}$$
 
We will pick the contour $C$ to enforce $f(t)e^{zt} \Bigg |_{C}=0$. We will do this by first determining the function $f$. Plugging this back into the formula $(*)$ yields
 
We will pick the contour $C$ to enforce $f(t)e^{zt} \Bigg |_{C}=0$. We will do this by first determining the function $f$. Plugging this back into the formula $(*)$ yields
Line 34: Line 34:
 
We have the freedom to choose $f$ and $C$. We will choose $f$ so that
 
We have the freedom to choose $f$ and $C$. We will choose $f$ so that
 
$$t^2f(t)-f'(t)=0.$$
 
$$t^2f(t)-f'(t)=0.$$
This is a simple differential equation whose solution [http://www.wolframalpha.com/input/?i=t^2f%28t%29-f%27%28t%29%3D0 is]
+
This is a simple differential equation with [http://www.wolframalpha.com/input/?i=t^2f%28t%29-f%27%28t%29%3D0 a solution]
$$f(t)=c e^{\frac{t^3}{3}}.$$
+
$$f(t)=e^{\frac{t^3}{3}}.$$
 
So we have derived
 
So we have derived
$$y(z)=\displaystyle\int_{C} e^{zt + \frac{t^3}{3}} dt.$$
+
$$y(z)=\displaystyle\int_{C} e^{-zt + \frac{t^3}{3}} dt.$$
 
To pick the contour $C$ note that the integrand of $y$ is an [[entire function]] and hence if $C$ is a simple closed curve we would have $y(z)=0$ for all $z \in \mathbb{C}$.  
 
To pick the contour $C$ note that the integrand of $y$ is an [[entire function]] and hence if $C$ is a simple closed curve we would have $y(z)=0$ for all $z \in \mathbb{C}$.  
  
 
The variable of the integral defining $y$ is $t$ and for $t \in \mathbb{C}$ with $|t|$ very large, the cubic term in the exponent dominates. Hence consider polar form $t=|t|e^{i\theta}$ and compute
 
The variable of the integral defining $y$ is $t$ and for $t \in \mathbb{C}$ with $|t|$ very large, the cubic term in the exponent dominates. Hence consider polar form $t=|t|e^{i\theta}$ and compute
 
$$e^{\frac{t^3}{3}} = \exp\left( \frac{|t|^3 e^{3i\theta}}{3} \right).$$
 
$$e^{\frac{t^3}{3}} = \exp\left( \frac{|t|^3 e^{3i\theta}}{3} \right).$$
Notice that the inequality $\mathrm{Re} \hspace{2pt} e^{3i\theta} \leq 0$ forces $\cos(3\theta)\leq 0$ yielding three sectors defined by $\theta$: [[File:Airysectors.png|200px]]
+
Notice that the inequality $\mathrm{Re} \hspace{2pt} e^{3i\theta} \leq 0$ forces $\cos(3\theta)\leq 0$ [http://www.wolframalpha.com/input/?i=cos%283*theta%29%3C0 yielding] three sectors defined by $\theta$: [[File:Airysectors.png|200px]]
 
$$-\dfrac{\pi}{2} \leq \theta \leq -\dfrac{\pi}{6},$$
 
$$-\dfrac{\pi}{2} \leq \theta \leq -\dfrac{\pi}{6},$$
 
$$\dfrac{\pi}{6} \leq \theta \leq \dfrac{\pi}{2},$$
 
$$\dfrac{\pi}{6} \leq \theta \leq \dfrac{\pi}{2},$$
 
$$\dfrac{9\pi}{6} \leq \theta \leq \dfrac{11\pi}{6}.$$
 
$$\dfrac{9\pi}{6} \leq \theta \leq \dfrac{11\pi}{6}.$$
  
Notice that the first two of these sectors includes the entire $y$-axis in the complex plane. Hence we will take our contour $C$ to be the $y$ axis directed from $-i\infty$ toward $+i\infty$. This yields finally
+
We will consider three contours $C_1,C_2,C_3$, where each contour $C_i$ has endpoints at complex $\infty$ in different sectors. Call the left sector $\gamma$, the upper-right sector $\beta$ and the lower-right sector $\alpha$. Let $C_1$ be oriented from sector $\alpha$ to sector $\beta$ (this sort of curve is labelled as "$C$" in the image above), $C_2$ from sector $\beta$ to sector $\gamma$, and $C_3$ from sector $\gamma$ to sector $\alpha$. By our analysis we have derived three solutions to Airy's equation:
 +
$$y_i(z) = \displaystyle\int_{C_i} e^{-zt + \frac{t^3}{3}} dt;i=1,2,3$$
 +
Since these functions satisfy a second order differential equation, it is impossible for them to be [[linearly independent]]. Now notice that we can compute
 +
$$\displaystyle\int_{C_1\cup C_2 \cup C_3} e^{-zt + \frac{t^3}{3}} dt = 0.$$
 +
Therefore
 +
$$y_1(z)+y_2(z)+y_3(z)=0.$$
 +
 
 +
Notice that the first two of these sectors includes the entire $y$-axis in the complex plane.0 Hence we will take our contour $C$ to be the $y$ axis directed from $-i\infty$ toward $+i\infty$. This yields finally
 
$$y(z)=\displaystyle\int_{-i\infty}^{i\infty} e^{zt+\frac{t^3}{3}} dt.$$
 
$$y(z)=\displaystyle\int_{-i\infty}^{i\infty} e^{zt+\frac{t^3}{3}} dt.$$
 
</div>
 
</div>

Revision as of 03:53, 16 January 2015

The Airy function $\mathrm{Ai}$ in the complex plane are given by $$\mathrm{Ai}(z) = \dfrac{1}{2\pi i} \displaystyle\int_{-i\infty}^{i\infty}\exp \left( \dfrac{t^3}{3} - zt \right) dt$$ and $$\mathrm{Bi}(x) = \dfrac{1}{\pi} \displaystyle\int_0^{\infty} \left[ e^{-\frac{t^3}{3} + xt} + \sin \left( \dfrac{t^3}{3}+xt \right) \right] dt.$$

Properties

Theorem: The function $\mathrm{Ai}$ is a solution to the differential equation $$y(z) - zy(z) = 0.$$

Proof: Suppose that $y$ has the form $$y(z) = \displaystyle\int_{C} f(t)e^{-zt} dt,$$ where $C$ is an as-of-yet undefined contour in the complex plane. Assuming that we may differentiate under the integral it is clear that $$y(z)=\displaystyle\int_{C} f(t)t^2 e^{-zt} dt.$$ Thus we plug this representation into the differential equation to get $$(*) \hspace{35pt} y(z)-zy(z) = \displaystyle\int_{C} (t^2-z)f(t)e^{-zt} dt = 0.$$ Now we integrate by parts to see $$\begin{array}{ll} \displaystyle\int_{C} zf(t)e^{-zt} dt &= -\displaystyle\int_{C} f(t) \dfrac{d}{dt} e^{-zt} dt \\ &= -f(t)e^{zt} \Bigg |_{C} + \displaystyle\int_{C} f'(t)e^{-zt} dt. \end{array}$$ We will pick the contour $C$ to enforce $f(t)e^{zt} \Bigg |_{C}=0$. We will do this by first determining the function $f$. Plugging this back into the formula $(*)$ yields $$\begin{array}{ll} 0 &= y(z) - zy(z) \\ &= f(t)e^{zt} \Bigg |_{C} + \displaystyle\int_{C} (t^2f(t)-f'(t))e^{zt} dt. \end{array}$$ We have the freedom to choose $f$ and $C$. We will choose $f$ so that $$t^2f(t)-f'(t)=0.$$ This is a simple differential equation with a solution $$f(t)=e^{\frac{t^3}{3}}.$$ So we have derived $$y(z)=\displaystyle\int_{C} e^{-zt + \frac{t^3}{3}} dt.$$ To pick the contour $C$ note that the integrand of $y$ is an entire function and hence if $C$ is a simple closed curve we would have $y(z)=0$ for all $z \in \mathbb{C}$.

The variable of the integral defining $y$ is $t$ and for $t \in \mathbb{C}$ with $|t|$ very large, the cubic term in the exponent dominates. Hence consider polar form $t=|t|e^{i\theta}$ and compute $$e^{\frac{t^3}{3}} = \exp\left( \frac{|t|^3 e^{3i\theta}}{3} \right).$$ Notice that the inequality $\mathrm{Re} \hspace{2pt} e^{3i\theta} \leq 0$ forces $\cos(3\theta)\leq 0$ yielding three sectors defined by $\theta$: Airysectors.png $$-\dfrac{\pi}{2} \leq \theta \leq -\dfrac{\pi}{6},$$ $$\dfrac{\pi}{6} \leq \theta \leq \dfrac{\pi}{2},$$ $$\dfrac{9\pi}{6} \leq \theta \leq \dfrac{11\pi}{6}.$$

We will consider three contours $C_1,C_2,C_3$, where each contour $C_i$ has endpoints at complex $\infty$ in different sectors. Call the left sector $\gamma$, the upper-right sector $\beta$ and the lower-right sector $\alpha$. Let $C_1$ be oriented from sector $\alpha$ to sector $\beta$ (this sort of curve is labelled as "$C$" in the image above), $C_2$ from sector $\beta$ to sector $\gamma$, and $C_3$ from sector $\gamma$ to sector $\alpha$. By our analysis we have derived three solutions to Airy's equation: $$y_i(z) = \displaystyle\int_{C_i} e^{-zt + \frac{t^3}{3}} dt;i=1,2,3$$ Since these functions satisfy a second order differential equation, it is impossible for them to be linearly independent. Now notice that we can compute $$\displaystyle\int_{C_1\cup C_2 \cup C_3} e^{-zt + \frac{t^3}{3}} dt = 0.$$ Therefore $$y_1(z)+y_2(z)+y_3(z)=0.$$

Notice that the first two of these sectors includes the entire $y$-axis in the complex plane.0 Hence we will take our contour $C$ to be the $y$ axis directed from $-i\infty$ toward $+i\infty$. This yields finally $$y(z)=\displaystyle\int_{-i\infty}^{i\infty} e^{zt+\frac{t^3}{3}} dt.$$

References

Tables of Weyl Fractional Integrals for the Airy Function